0 Daumen
288 Aufrufe

Aufgabe:

F(t) = \( \int\limits_{0}^{\infty} \) \( \frac{sin(x)}{x} \) \( e^{-xt} \) dx


Problem/Ansatz:

Es soll gezeigt werden, dass F wohldefiniert ist $$(F(t) < ∞ ∀t≥0)$$

Ich weiß aber nicht so recht, wie ich vorgehen soll (prinzipiell müsste ich doch zeigen, dass das Ergebnis des Integrals nicht von der Wahl der Stammfunktion abhängt - oder?)

Avatar von

2 Antworten

+1 Daumen
 
Beste Antwort

Es muss gezeigt werden, dass für \(t\geq 0\) das Integral konvergiert.

Da \(\lim_{x\to 0}\frac {\sin x}{x} =1\), lässt sich der Integrand stetig in \(x=0\) fortsetzen. Damit wissen wir schon mal, dass \(\int_0^{\color{blue}{1}}\frac {\sin x}{x}e^{-xt}dx\) konvergiert.

Für \(t>0\) gilt

\(\int_1^{\infty} \left|\frac {\sin x}{x}e^{-xt}\right|dx \leq \int_1^{\infty}e^{-xt}dx <\infty\)

Für \(t=0\) ist \(e^{-tx}=1\). Also gilt mit partieller Integration

\(\int_1^{\infty} \frac {\sin x}{x} dx = \left. -\frac {\cos x}{x} \right|_{1}^{\infty}-\int_1^{\infty} \frac {\cos x}{x^2} dx <\infty\).

Somit is \(F(t)\) für \(t\geq 0\) wohldefiniert, da jedem \(t\geq 0\) ein eindeutiger endlicher reeller Wert zugeordnet wird.

Avatar von 10 k

Vielen Dank für deine schnelle Hilfe ☺

+2 Daumen

Aloha :)$$F(t)=\int\limits_0^\infty\frac{\sin x}{x}\,e^{-xt}\,dx\quad;\quad t\ge0$$

Ich würde das Integral einfach ausrechnen. Dazu bestimme zuerst \(F'(t)\) und integriere das Ergebnis anschließend nach \(dt\). Mit Hilfe der Leibniz-Regel für Parameterintegrale gilt:$$F'(t)=\int\limits_0^\infty\frac{\partial}{\partial t}\left(\frac{\sin x}{x}\,e^{-xt}\right)dx=\int\limits_0^\infty\frac{\sin x}{x}\cdot(-x)\,e^{-xt}\,dx=\int\limits_0^\infty\underbrace{(-\sin x)}_{=u'}\cdot \underbrace{e^{-xt}}_{=v}\,dx$$

Das unbestimmte Integral bestimmen wir mittels partieller Integration:$$I(x)=\int\underbrace{(-\sin x)}_{=u'}\cdot \underbrace{e^{-xt}}_{=v}\,dx=\underbrace{\cos x}_{=u}\cdot \underbrace{e^{-xt}}_{=v}-\int\underbrace{\cos x}_{=u}\cdot \underbrace{(-te^{-xt})}_{=v'}\,dx$$$$\phantom{I(t)}=\cos x\cdot e^{-xt}+t\int\underbrace{\cos x}_{=p'}\cdot \underbrace{e^{-xt}}_{=q}\,dx$$$$\phantom{I(t)}=\cos x\cdot e^{-xt}+t\left(\underbrace{\sin x}_{=p}\cdot \underbrace{e^{-xt}}_{=q}-\int\underbrace{\sin x}_{=p}\cdot\underbrace{(-te^{-xt})}_{=q'}\,dx\right)$$$$\phantom{I(t)}=\cos x\cdot e^{-xt}+t\sin x\cdot e^{-xt}-t^2\underbrace{\int(-\sin x)\cdot e^{-xt}\,dx}_{=I(x)}$$

Wir bringen alle Terme mit \(I(x)\) auf die linke Seite:$$I(x)+t^2\,I(x)=e^{-xt}(\cos x+t\sin x)\quad\implies\quad I(x)=\frac{e^{-xt}(\cos x+t\sin x)}{1+t^2}$$

Das bedeutet für die Ableitung \(F'(t)\) von oben:$$\small F'(t)=I(x\to\infty)-I(x=0)=\underbrace{\lim\limits_{x\to\infty}\left(\frac{e^{-xt}(\cos x+t\sin x)}{1+t^2}\right)}_{=0}-\left(\frac{1}{1+t^2}\right)=-\frac{1}{1+t^2}$$

Integration liefert (fast) das gesuchte Integral:$$F(t)=-\arctan(t)+C$$

Es fehlt noch die Integrationskonstante \(C\):$$\small-\arctan(t)+C=F(t)\stackrel!=\int\limits_0^\infty\frac{\sin x}{x}\,e^{-xt}\,dx\stackrel{(t\to\infty)}{\implies}-\frac\pi2+C=\int\limits_0^\infty\frac{\sin x}{x}\cdot0\,dx=0\implies C=\frac\pi2$$

Damit erhalten wir schließlich:$$F(t)=\int\limits_0^\infty\frac{\sin x}{x}\,e^{-xt}\,dt=\frac\pi2-\arctan(t)$$

Avatar von 149 k 🚀

Auch dir vielen Dank . Das Integral explizit zu berechnen, wurde bei mir in einer extra Aufgabe gestellt (\( \int\limits_{0}^{\infty} \) \( \frac{sin(x)}{x} \) dx = \( \frac{π}{2} \)).

Jetzt habe ich aber dafür eine gute Kontrolle - habe nur statt  $$ - arctan(t) $$ den Ausdruck $$ -tan^{-1}(t) $$ verwendet (läuft ja aber aufs selbe hinaus, wenn man Null einsetzt ☺).

Das ist dasselbe. Du kannst aber etwas eleganter noch schreiben:$$\frac\pi2-\arctan(t)=\arctan\left(\frac1t\right)$$

Danke für diesen Hinweis

Ein anderes Problem?

Stell deine Frage

Willkommen bei der Mathelounge! Stell deine Frage einfach und kostenlos

x
Made by a lovely community